X-Git-Url: http://www.chiark.greenend.org.uk/ucgi/~ian/git?a=blobdiff_plain;f=article.tex;h=a0adacbdee9b8d02bb1f2421f8158b1a43b505b2;hb=d3266383df1c90e7ff2f28acd9ebbd217ffc6d55;hp=023a18a9340c7670e3ee89011177eb9010165e69;hpb=1b7bb2a25de86ded197df472caad027f0111a449;p=topbloke-formulae.git diff --git a/article.tex b/article.tex index 023a18a..a0adacb 100644 --- a/article.tex +++ b/article.tex @@ -81,7 +81,8 @@ \newcommand{\Largenexists}{\mathop{\hbox{\Large$\nexists$}}} \newcommand{\qed}{\square} -\newcommand{\proof}[1]{{\it Proof.} #1 $\qed$} +\newcommand{\proofstarts}{{\it Proof:}} +\newcommand{\proof}[1]{\proofstarts #1 $\qed$} \newcommand{\gathbegin}{\begin{gather} \tag*{}} \newcommand{\gathnext}{\\ \tag*{}} @@ -262,7 +263,7 @@ XXX proof TBD. \subsection{No Replay for Merge Results} -If we are constructing $C$, given +If we are constructing $C$, with, \gathbegin \mergeof{C}{L}{M}{R} \gathnext @@ -270,7 +271,7 @@ If we are constructing $C$, given \gathnext R \le C \end{gather} -No Replay is preserved. {\it Proof:} +No Replay is preserved. \proofstarts \subsubsection{For $D=C$:} $D \isin C, D \le C$. OK. @@ -424,7 +425,7 @@ Merge Results applies. $\qed$ \subsection{Desired Contents} \[ D \isin C \equiv [ D \notin \pry \land D \isin L ] \lor D = C \] -{\it Proof.} +\proofstarts \subsubsection{For $D = C$:} @@ -473,6 +474,7 @@ Merge commits $L$ and $R$ using merge base $M$ ($M < L, M < R$): \gathnext \mergeof{C}{L}{M}{R} \end{gather} +We will occasionally use $X,Y$ s.t. $\{X,Y\} = \{L,R\}$. \subsection{Conditions} @@ -527,8 +529,38 @@ $\qed$ \subsection{Coherence and patch inclusion} -xxx tbd +Need to determine $C \haspatch P$ based on $L,M,R \haspatch P$. +This involves considering $D \in \py$. + +\subsubsection{For $L \nothaspatch P, R \nothaspatch P$:} +$D \not\isin L \land D \not\isin R$. $C \not\in \py$ (otherwise $L +\in \py$ ie $L \haspatch P$ by Tip Self Inpatch). So $D \neq C$. +Applying $\merge$ gives $D \not\isin C$ i.e. $C \nothaspatch P$. + +\subsubsection{For $L \haspatch P, R \haspatch P$:} +$D \isin L \equiv D \le L$ and $D \isin R \equiv D \le R$. +(Likewise $D \isin X \equiv D \le X$ and $D \isin Y \equiv D \le Y$.) + +Consider $D = C$: $D \isin C$, $D \le C$, OK for $C \haspatch P$. + +For $D \neq C$: $D \le C \equiv D \le L \lor D \le R + \equiv D \isin L \lor D \isin R$. +(Likewise $D \le C \equiv D \le X \lor D \le Y$.) + +Consider $D \neq C, D \isin X \land D \isin Y$: +By $\merge$, $D \isin C$. Also $D \le X$ +so $D \le C$. OK for $C \haspatch P$. + +Consider $D \neq C, D \not\isin X \land D \not\isin Y$: +By $\merge$, $D \not\isin C$. +And $D \not\le X \land D \not\le Y$ so $D \not\le C$. +OK for $C \haspatch P$. + +Remaining case, wlog, is $D \not\isin X \land D \isin Y$. +$D \not\le X$ so $D \not\le M$ so $D \not\isin M$. +Thus by $\merge$, $D \isin C$. And $D \le Y$ so $D \le C$. +OK for $C \haspatch P$. -xxx need to finish merge +So indeed $L \haspatch P \land R \haspatch P \implies C \haspatch P$. \end{document}